1
$\begingroup$

I hope this is not too elementary.

Let $B: V\ \times V \to W$ be a skew-symmetric bilinear map where $V$, $W$ are finite dimensional real vector spaces. Assume that $B (u, v)$ is never zero for linearly independent $u$, $v\in V$.

What can we say about the dimensions of $V$ and $W$?

An elementary estimate is

$$ \dim W \geq \dim V - 1 $$

In fact, for $u\in V$, $u\neq0$, the linear map $B_u : V\to W$, $B_u(v)=B(u,v)$, is injective on a codimension $1$ subspace of $V$. Can the above estimate be improved? Recall that the image of a bilinear map does not need to be a subspace.

$\endgroup$

1 Answer 1

3
$\begingroup$

A better bound is $\dim W\geq 2\dim V-3$. This is obtained as follows. In the projective space $\mathbb{P}(\wedge^2V)$, the set of decomposable bivectors is the Grassmannian $\mathbb{G}(2,V)$, of dimension $2\dim V-4$. The bilinear map $B$ induces a linear map $b:\wedge^2V\rightarrow W$, and the hypothesis is $\mathbb{P}(\mathrm{Ker}\, b)\cap \mathbb{G}(2,V)=\varnothing$. This implies $2\dim V-4<\mathrm{codim}\, \mathrm{Ker}\, b=\dim \mathrm{Im}\, b$, hence the result.

$\endgroup$
6
  • $\begingroup$ Simple and smart, thanks! In fact, the better bound suffices for what I need. $\endgroup$ Jan 9, 2015 at 16:00
  • $\begingroup$ This is way late, but I think the argument works over $\mathbb C$ but not over $\mathbb R$, which is what I wanted. The obvious idea of complexifying $B$ does not seem to preserve the assumption on $B$. $\endgroup$ Feb 21, 2015 at 14:07
  • $\begingroup$ I think this is a counter-example: $\mathbb G(2,\mathbb R^4)$ has dim $4$ and embedds in $\mathbb P(\Lambda^2\mathbb R^4)$ which has dim $5$, defined $\endgroup$ Feb 21, 2015 at 14:09
  • $\begingroup$ the equation $a_{12}a_{34}-a_{13}a_{24}+a_{14}a_{23}=0$. Construct a linear map $\endgroup$ Feb 21, 2015 at 14:10
  • $\begingroup$ $b:\Lambda^2\mathbb R^4\to W^3$ whose kernel is $a_{12}=a_{34}$, $a_{14}=a_{23}$, $a_{13}=-a_{24}$, and consider its projectivization $U^2 \subset \mathbb P (\Lambda^2\mathbb R^4)$. Then $U^2$ does not meet $\mathbb G(2,\mathbb R^4)$. On the other hand, $2\dim V - 3 = 5 > 3 = \dim W$. $\endgroup$ Feb 21, 2015 at 14:14

Your Answer

By clicking “Post Your Answer”, you agree to our terms of service and acknowledge you have read our privacy policy.

Not the answer you're looking for? Browse other questions tagged or ask your own question.